Zweite Ableitung des Dirac-Delta-Ausdrucks

Ich bin auf den Ausdruck gestoßen

F ( X ) δ ( X A ) δ ( X A ) D X
wobei die Primzahl die Ableitung darstellt.

Normalerweise würde ich bei Derivaten der Dirac-Delta-Verteilung teilweise integrieren, aber hier drehe ich mich immer im Kreis. Was ich versucht habe ist

F δ δ = ( F δ ) ' δ ' = F ' δ δ ' F δ ' δ ' = F ' δ δ ' + ( F δ ' ) ' δ = F ' δ δ ' + F ' δ ' δ + F δ δ = F δ δ
oder wenn ich den anderen Term für die zweite partielle Integration nehme
F δ δ = ( F δ ) ' δ ' = ( F δ ) δ = F δ δ + 2 F ' δ ' δ + F δ δ 0 = F δ δ + 2 F ' δ ' δ
die ich auch durch elementare partielle Integration hätte bekommen können F δ 2 .

Welche weiteren Möglichkeiten habe ich? Quadrate von Deltafunktionen etc. sind kein Problem.

Wenden Sie einfach die reguläre Regel an, um eine Deltafunktion zu integrieren, und ersetzen Sie sie überall sonst X = A . Ihre integralen Erträge F ( A ) δ ( 0 ) .
@JamalS Aber Werte einer Ableitung der Deltaverteilung sind nicht definiert. Nur der δ ohne Derivate ausgewertet werden können.
Können Sie den spezifischen Kontext angeben, in dem dieses Integral entsteht?
Die doppelt abgeleitete Deltafunktion entsteht in Theorien mit höheren Dimensionen, wenn man die schleifeninduzierten FI-Terme berechnet. Wenn Sie diesen FI-Term an einen Bran-Skalar koppeln und den FI-Term nicht durch andere Mittel (z. B. Hintergrundflüsse) kompensieren möchten, erscheint eine Kombination wie die beschriebene in der Aktion. Um die effektive 4D-Aktion abzuleiten, muss ich über die internen Dimensionen integrieren, was mir das obige Integral gibt.
Mir ist keine vernünftige Definition des Quadrats einer Delta-Funktion bekannt. Wie definieren Sie es?
Es ist nicht so sehr eine vernünftige Definition, aber die entstehenden Unendlichkeiten können so umgeschrieben werden, dass sie divergierende Teile von Termen höherer Ordnung aufheben. δ 2 ist immer noch nicht sehr genau definiert, aber ich kann das loswerden δ ( 0 ) das ergibt sich und für die meisten Physiker reicht das aus - das ist eigentlich der Grund, warum ich das hier und nicht auf math.stackexchange.com stelle
Zum Produkt von Dirac-Delta-Verteilungen: mathoverflow.net/q/48067 , physical.stackexchange.com/q/47934/2451 und Links darin.
Nur fürs Protokoll, mathematische Ableitungen der Delta-Funktionen (oder jeder anderen Verteilung) sind gut definiert (als Verteilungen): δ ' ( X ) ist die Verteilung so, dass δ ' ( X ) F ( X ) D X = F ' ( 0 ) .
@yuggib: Ja, ich denke im Allgemeinen F ( X ) δ ( N ) ( X A ) D X = ( 1 ) N F ( N ) ( A ) .
Ich verstehe, dass das Integral von -∞ bis +∞ reicht. Dann ist es sinnvoll, die Variable u = x - a zu ändern. Also hast du ∫f(u+a)δ(u)δ′′(u)du . Im Übrigen sehe ich kein mögliches Entweichen von δ′′, ich habe es auch versucht. Lassen Sie uns also Ihren anfänglichen Integranden direkt integrieren. Sie erhalten ∫f(u+a)δ(u)δ′′(u)du = f(a)δ′′(0). Leider ist δ′′(0) = −∞ .
Kommentar zur Frage (v2): Den Kommentar von @JamalS wiederholen: Erwägen Sie das Hinzufügen von Referenzen, um nützliche und gezielte Antworten zu erhalten.
Ich denke, Sie können Verteilungen nur dann multiplizieren, wenn die Singularitäten jeder der Verteilungen nicht übereinander liegen. Die Tatsache, dass Sie zwei Verteilungen mit Singularitäten am selben Punkt haben, lässt den Anschein erwecken, dass dies schlecht definiert ist.

Antworten (1)

Ich habe nur mit Dingen gespielt und das bekommen, lassen Sie es mich wissen, wenn Sie es für eine nützliche Idee halten (oder wenn mir etwas fehlt, was dies nutzlos macht).

Bezeichnen δ k ( X ) = 1 k π e ( X / k ) 2 . Sie erhalten das bekannte (völlig mathematisch strenge) Ergebnis:

lim k 0 F ( X ) δ k ( X ) D X = F ( 0 ) für gut erzogen genug F .

Schauen wir uns also an:

lim k 0 ( C 0 + C 1 X + C 2 X 2 + ) δ k ( X ) δ k ( X ) D X = F ( 0 )

Unter Verwendung der bekannten Technik des Beweises von Mathematica:

delta[x_] := 1/(k Sqrt[ Pi]) E^(-(x/k)^2);
g[x_] = FullSimplify[delta''[x] delta[x] x^n];
Integrate[g[x], {x, -Infinity, Infinity}, Assumptions -> {Element[n, Integers], a > 0}]

Ich bekomme das Ergebnis:

X N δ k ( X ) δ k ( X ) D X = 2 N 2 3 2 ( ( 1 ) N + 1 ) ( N 1 ) k N 3 Γ ( N + 1 2 ) π

Als k 0 + , der Wert ist Null für N = 1 , 3 , 4 , 5 , 6 , 7 , und divergiert für N = 0 , 2 , wie 1 2 π k 3 für die N = 0 Fall und dergleichen 1 4 k 2 π für die N = 2 Fall.

Gute Idee zum anschauen F ( X ) = X N ! Tatsächlich für N > 2 wir haben X N δ = 0 , sodass das Integral sinnvoll und 0 ist. Außerdem haben wir X 0 δ = δ , X 1 δ = 2 δ ' Und X 2 δ = δ . Beachten Sie, dass Ihr Ergebnis, für das wir 0 erhalten N = 1 , sowie die Wachstumsreihenfolge, sind Artefakte der Sequenz, zu der Sie sich entschieden haben, zu konvergieren δ .
@doetoe Wie können Sie sicher sein, dass es sich nur um Artefakte handelt? (obwohl ich denke, du hast recht, da die 2 π scheint zu spezifisch). Der Prozess funktioniert genau (keine Artefakte) für δ ( N ) ( X )
@doetoe eigentlich, streich das, ich denke, es ist ein bisschen offensichtlich, dass sie Artefakte sind, und nur die k = 0 Verhalten (das nicht existiert) ist der wichtige Teil.